K
Khách

Hãy nhập câu hỏi của bạn vào đây, nếu là tài khoản VIP, bạn sẽ được ưu tiên trả lời.

18 tháng 12 2018

Ngc dấu r bạn

5 tháng 2 2020

Áp dụng BĐT Cô-si cho 3 số dương, ta có :

\(\frac{1}{a\left(a+b\right)}+\frac{1}{b\left(b+c\right)}+\frac{1}{c\left(a+c\right)}\ge3\sqrt[3]{\frac{1}{abc\left(a+b\right)\left(b+c\right)\left(a+c\right)}}\)

Cần chứng minh : \(\sqrt[3]{\frac{1}{abc\left(a+b\right)\left(b+c\right)\left(a+c\right)}}\ge\frac{9}{2\left(a+b+c\right)^2}\)

hay \(8\left(a+b+c\right)^6\ge729abc\left(a+b\right)\left(b+c\right)\left(a+c\right)\)

Thật vậy, ta có : \(\left(a+b+c\right)^3\ge\left(3\sqrt[3]{abc}\right)^3=27abc\)

\(8\left(a+b+c\right)^3=\left(2\left(a+b+c\right)\right)^3=\left(a+b+b+c+a+c\right)^3\)

\(\ge\left(3\sqrt[3]{\left(a+b\right)\left(b+c\right)\left(a+c\right)}\right)^3=27\left(a+b\right)\left(b+c\right)\left(a+c\right)\)

Nhân từng vế 2 bất đẳng thức trên, ta được đpcm

Dấu "=" xảy ra khi a = b = c 

Vậy ...

5 tháng 2 2020

2. Áp dụng BĐT Cô-si cho 3 số không âm, ta có : 

\(B\ge3\sqrt[3]{\sqrt{\left(a^3+b^3+1\right)\left(b^3+c^3+1\right)\left(a^3+c^3+1\right)}}\)

Ta có : \(a^3+b^3+1\ge3\sqrt[3]{a^3b^3}=3ab\Rightarrow\sqrt{a^3+b^3+1}\ge\sqrt{3ab}\)

Tương tự : ....

\(\Rightarrow\sqrt{\left(a^3+b^3+1\right)\left(b^3+c^3+1\right)\left(c^3+a^3+1\right)}\ge\sqrt{27a^2b^2c^2}=\sqrt{27}\)

\(\Rightarrow B\ge3\sqrt[3]{\sqrt{27}}=3\sqrt{3}\)

Vậy GTNN của B là \(3\sqrt{3}\)khi a = b = c = 1

từ giả thiết, ta có \(\dfrac{1}{xy}+\dfrac{1}{yz}+\dfrac{1}{zx}=1\) đặt \(\left(\dfrac{1}{xy};\dfrac{1}{yz};\dfrac{1}{zx}\right)=\left(a;b;c\right)\Rightarrow a+b+c=1\) =>\(\left(\dfrac{ac}{b};\dfrac{ab}{c};\dfrac{bc}{a}\right)=\left(\dfrac{1}{x^2};\dfrac{1}{y^2};\dfrac{1}{z^2}\right)\) ta có...
Đọc tiếp

từ giả thiết, ta có \(\dfrac{1}{xy}+\dfrac{1}{yz}+\dfrac{1}{zx}=1\)

đặt \(\left(\dfrac{1}{xy};\dfrac{1}{yz};\dfrac{1}{zx}\right)=\left(a;b;c\right)\Rightarrow a+b+c=1\) =>\(\left(\dfrac{ac}{b};\dfrac{ab}{c};\dfrac{bc}{a}\right)=\left(\dfrac{1}{x^2};\dfrac{1}{y^2};\dfrac{1}{z^2}\right)\)

ta có VT=\(\dfrac{1}{\sqrt{1+\dfrac{1}{x^2}}}+\dfrac{1}{\sqrt{1+\dfrac{1}{y^2}}}+\dfrac{1}{\sqrt{1+\dfrac{1}{z^1}}}=\sqrt{\dfrac{1}{1+\dfrac{ac}{b}}}+\sqrt{\dfrac{1}{1+\dfrac{ab}{c}}}+\sqrt{\dfrac{1}{1+\dfrac{bc}{a}}}\)

=\(\dfrac{1}{\sqrt{\dfrac{b+ac}{b}}}+\dfrac{1}{\sqrt{\dfrac{a+bc}{a}}}+\dfrac{1}{\sqrt{\dfrac{c+ab}{c}}}=\sqrt{\dfrac{a}{\left(a+b\right)\left(a+c\right)}}+\sqrt{\dfrac{b}{\left(b+c\right)\left(b+a\right)}}+\sqrt{\dfrac{c}{\left(c+a\right)\left(c+b\right)}}\)

\(\le\sqrt{3}\sqrt{\dfrac{ac+ab+bc+ba+ca+cb}{\left(a+b\right)\left(b+c\right)\left(c+a\right)}}=\sqrt{3}.\sqrt{\dfrac{2\left(ab+bc+ca\right)}{\left(a+b\right)\left(b+c\right)\left(c+a\right)}}\)

ta cần chứng minh \(\sqrt{\dfrac{2\left(ab+bc+ca\right)}{\left(a+b\right)\left(b+c\right)\left(c+a\right)}}\le\dfrac{3}{2}\Leftrightarrow\dfrac{2\left(ab+bc+ca\right)}{\left(a+b\right)\left(b+c\right)\left(c+a\right)}\le\dfrac{9}{4}\Leftrightarrow8\left(ab+bc+ca\right)\le9\left(a+b\right)\left(b+c\right)\left(c+a\right)\)

<=>\(8\left(a+b+c\right)\left(ab+bc+ca\right)\le9\left(a+b\right)\left(b+c\right)\left(c+a\right)\) (luôn đúng )

^_^

0
5 tháng 1 2021

Cho a,b,c là các số thực dương thỏa mãn a+b+c = 3

Chứng minh rằng với mọi k > 0 ta luôn có....

5 tháng 1 2021

.

Cho a,b,c là các số thực dương thỏa mãn a+b+c = 3

Chứng minh rằng với mọi k > 0 ta luôn có

24 tháng 7 2023

Đặt \(\left\{{}\begin{matrix}a^2-bc=x\\b^2-ca=y\\c^2-ab=z\end{matrix}\right.\)

\(\Rightarrow x+y+z\ge0\)

\(\)Đẳng thức cần c/m trở thành: \(x^3+y^3+z^3\ge3xyz\left(1\right)\)

Áp dụng Bất đẳng thức AM-GM cho 3 số x,y,z, ta có:

\(x^3+y^3+z^3\ge3\sqrt[3]{x^3.y^3.z^3}=3xyz\)

=> Đẳng thức (1) luôn đúng với mọi x

Dấu = xảy ra khi: x=y=z hay \(a^2-bc=b^2-ca=c^2-ab\)

và \(a^2+b^2+c^2-\left(ab+bc+ca\right)=0\)\(\Rightarrow a=b=c\)

19 tháng 1 2020

ez mà :)))

19 tháng 1 2020

bạn ơi, hình như bạn nhớ nhầm rồi đấy, ko có HĐT đó đâu, mà có HĐT thức ấy nhưng a+b+c = 0  nữa cơ

AH
Akai Haruma
Giáo viên
27 tháng 6 2020

Nguyễn Xuân Đình Lực:

mình ghi rõ trên rùi, sắp xếp theo thứ tự luôn cho dễ nhìn kìa bạn:

Cặp 1: $a^3b$ và $abc^2$ tạo ra $a^2bc$

Cặp 2: $b^3c$ và $bca^2$ tạo ra $b^2ca$

Cặp 3: $c^3a$ và $cab^2$ tạo ra $c^2ab$

AH
Akai Haruma
Giáo viên
27 tháng 6 2020

Lời giải:

Ba số thực $a,b,c$ cần có thêm điều kiện không âm mới đúng.

BĐT cần chứng minh tương đương với:

$ab^3+bc^3+ca^3+2abc(a+b+c)\leq a^3b+b^3c+c^3a+ab^3+bc^3+ca^3+abc(a+b+c)$

$\Leftrightarrow abc(a+b+c)\leq a^3b+b^3c+c^3a(*)$

Áp dụng BĐT Bunhiacopxky:

$(a^3b+b^3c+c^3a)(abc^2+bca^2+cab^2)\geq (a^2bc+b^2ca+c^2ab)^2$

$\Rightarrow a^3b+b^3c+c^3a\geq abc(a+b+c)$

BĐT $(*)$ đúng nên ta có đpcm.

Dấu "=" xảy ra khi $a=b=c$

27 tháng 6 2020

a,b,c>0 

\(VP-VT=a^3b+b^3c+c^3a-abc\left(a+b+c\right)=abc\Sigma\frac{\left(a-b\right)^2}{a}\ge0\)